Mathcenter Forum  

Go Back   Mathcenter Forum > คณิตศาสตร์โอลิมปิก และอุดมศึกษา > คณิตศาสตร์อุดมศึกษา
สมัครสมาชิก คู่มือการใช้ รายชื่อสมาชิก ปฏิทิน ข้อความวันนี้

ตั้งหัวข้อใหม่ Reply
 
เครื่องมือของหัวข้อ ค้นหาในหัวข้อนี้
  #16  
Old 04 มกราคม 2006, 08:23
nongtum's Avatar
nongtum nongtum ไม่อยู่ในระบบ
ผู้พิทักษ์กฎทั่วไป
 
วันที่สมัครสมาชิก: 10 เมษายน 2005
ข้อความ: 3,246
nongtum is on a distinguished road
Post

อ้างอิง:
ข้อความเดิมของคุณ M@gpie:
จงแยกตัวประกอบของ
\(z^7+z^5+z^4+z^3+z^2+1\)
\(s^9+s^6+s^3+s^2+1\)
ข้อย่อยแรก จับคู่ใหม่แล้วแยกตัวประกอบทีละคู่จะได้ $z^7+z^5 +z^4+z^3+z^2+1=(z+1)(z^2-z+1)(z^2+z+1)^2$
ข้อย่อยที่สอง โดยใช้การทดสอบแบบด้านบน($s^5=1$) จะได้ $s^4+s^3+s^2+s+1$ เป็นตัวประกอบ นั่นคือ $s^9+s^6+s^3+s^2+1=(s^5-s^4+s^2-s+1)(s^4+s^3+s^2+s+1)$
ทั้งสอง factor ไม่สามารถแยกตัวประกอบต่อได้ ตรวจสอบได้โดย Eisenstein-Criterion (factor ตัวหลังเป็น cyclotomic polynomial สำหรับจำนวนเฉพาะ 5 ซึ่งแยกตัวประกอบต่อไม่ได้)
__________________
คนไทยร่วมใจอย่าใช้ภาษาวิบัติ
ฝึกพิมพ์สัญลักษณ์สักนิด ชีวิต(คนตอบและคนถาม)จะง่ายขึ้นเยอะ (จริงๆนะ)

Stay Hungry. Stay Foolish.
ตอบพร้อมอ้างอิงข้อความนี้
  #17  
Old 04 มกราคม 2006, 19:44
M@gpie's Avatar
M@gpie M@gpie ไม่อยู่ในระบบ
ลมปราณไร้สภาพ
 
วันที่สมัครสมาชิก: 09 ตุลาคม 2003
ข้อความ: 1,227
M@gpie is on a distinguished road
Post

เยี่ยมเลยครับ ข้อแรกของผมยังใช้วิธีจับคู่ได้อยู่
แต่ข้อสอง กำลังพยายามหาวิธีที่ไม่ต้องใช้วิธีทดสอบแบบนี้ คุณ nongtum หรือท่านอื่นๆ เห็นว่ามีทางไปได้ป่ะคับ ??
ผมกำลังพยายามไม่พึ่งวิธีนี้ แหะๆๆ เพราะ เหมือนโจทย์มันเฉพาะสุดๆ ต้องเป๊ะ ถึงจะได้ แต่ก็เป็นเทคนิค ที่ผมเพิ่งค้นพบ (หรือคนอื่นรู้อยู่แล้วหว่า??)
__________________
PaTa PatA pAtA Pon!
ตอบพร้อมอ้างอิงข้อความนี้
  #18  
Old 04 มกราคม 2006, 22:03
warut warut ไม่อยู่ในระบบ
กระบี่ไร้สภาพ
 
วันที่สมัครสมาชิก: 24 พฤศจิกายน 2001
ข้อความ: 1,627
warut is on a distinguished road
Post

ยังไม่เคลียร์ครับว่าจะทดสอบ irreducibility ของ \(s^5-s^4+s^2-s+1\) ด้วย Eisenstein's criterion ได้อย่างไร
ตอบพร้อมอ้างอิงข้อความนี้
  #19  
Old 14 มกราคม 2006, 00:13
nooonuii nooonuii ไม่อยู่ในระบบ
ผู้พิทักษ์กฎทั่วไป
 
วันที่สมัครสมาชิก: 25 พฤษภาคม 2001
ข้อความ: 6,408
nooonuii is on a distinguished road
Post

สวัสดีครับทุกท่าน หายไปสอบมาครับ ตอนนี้สอบเสร็จละ เลยมาเฉลยโจทย์ที่ตั้งไว้

1.(อสมการ) ให้ a,b,c > 0 จงพิสูจน์ว่า
\[ \frac{a}{\sqrt{7a^2 + b^2 + c^2 }} + \frac{b}{\sqrt{a^2 + 7b^2 + c^2 }} + \frac{c}{\sqrt{a^2 + b^2 + 7c^2 }} \leq 1 \]

ข้อนี้ผมคิดคล้ายคุณ passer-by ครับ แต่จบต่างกันนิดนึงเพราะผมใช้ AM-GM

My Solution: โดยอสมการโคชีจะได้ว่า
\( 7a + b + c = \sqrt{ 7 }\cdot \sqrt{ 7 }a + 1\cdot b + 1\cdot c \leq \sqrt{ 7+1+1 }\sqrt{ 7a^2+b^2+c^2 } = 3\sqrt{ 7a^2+b^2+c^2 } \)

ในทำนองเดียวกัน

\( a + 7b + c \leq 3\sqrt{ a^2 + 7b^2 + c^2 }\)
\( a + b + 7c \leq 3\sqrt{ a^2 + b^2 + 7c^2 } \)

ดังนั้น LHS \( \leq \displaystyle{\frac{3a}{7a+b+c} +\frac{3b}{a+7b+c} +\frac{3c}{a+b+7c}} \)
ต่อไปจะพิสูจน์ว่า
\( \displaystyle{\frac{3a}{7a+b+c} +\frac{3b}{a+7b+c} +\frac{3c}{a+b+7c} \leq 1} \)

ให้ \( x = 7a + b + c, y = a + 7b + c, z = a + b + 7c \)
จะได้ว่า \( \displaystyle{ 3a = \frac{8x - y - z}{18}, 3b = \frac{8y - z - x}{18}, 3c = \frac{8z - x - y}{18} } \)
ดังนั้น LHS = \( \displaystyle{ \frac{1}{18}\{(8 - \frac{y}{x}-\frac{z}{x}) + (8 - \frac{z}{y} - \frac{x}{y}) + (8 - \frac{x}{z} - \frac{y}{z}) \} = \frac{1}{18}\{24 - (\frac{y}{x} + \frac{z}{x} + \frac{z}{y} + \frac{x}{y} + \frac{x}{z} + \frac{y}{z}) \} \leq \frac{1}{18}(24 - 6) = 1} \)

__________________
site:mathcenter.net คำค้น

14 มกราคม 2006 00:19 : ข้อความนี้ถูกแก้ไขแล้ว 1 ครั้ง, ครั้งล่าสุดโดยคุณ nooonuii
ตอบพร้อมอ้างอิงข้อความนี้
  #20  
Old 14 มกราคม 2006, 05:36
passer-by passer-by ไม่อยู่ในระบบ
ผู้พิทักษ์กฎทั่วไป
 
วันที่สมัครสมาชิก: 11 เมษายน 2005
ข้อความ: 1,442
passer-by is on a distinguished road
Post

แล้วไม่เฉลยข้อ 2 เหรอครับ
__________________
เกษียณตัวเอง ปลายมิถุนายน 2557 แต่จะกลับมาเป็นครั้งคราว
ตอบพร้อมอ้างอิงข้อความนี้
  #21  
Old 14 มกราคม 2006, 06:03
warut warut ไม่อยู่ในระบบ
กระบี่ไร้สภาพ
 
วันที่สมัครสมาชิก: 24 พฤศจิกายน 2001
ข้อความ: 1,627
warut is on a distinguished road
Smile

ผมเชื่อมั่นว่าคุณ nooonuii คงต้องมาเฉลยจนหมดล่ะครับ แต่คงต้องใช้เวลามากหน่อย
ตอบพร้อมอ้างอิงข้อความนี้
  #22  
Old 14 มกราคม 2006, 10:40
Punk Punk ไม่อยู่ในระบบ
ลมปราณบริสุทธิ์
 
วันที่สมัครสมาชิก: 10 เมษายน 2005
ข้อความ: 108
Punk is on a distinguished road
Post

ข้อ 2 (ร่วมด้วยช่วยกัน (มั่ว) )

กรณี $n=0,\pm1$ พิสูจน์ได้ไม่ยากโดยอสมการ Cauchy สมมติ $|n|\geq2$

ให้ $n=\varepsilon k$ โดย $\varepsilon=-1,1$ แทนเครื่องหมายของ $n$ และ $k\geq2$ เป็นจำนวนนับ โดยอสมการ Holder กรณีทั่วไป
$$
\left(\frac{a^{\varepsilon k}}{b+1}+\frac{b^{\varepsilon k}}{c+1}
+\frac{c^{\varepsilon k}}{a+1}\right)((b+1)+(c+1)+(a+1))3\cdots3\geq(a^\varepsilon+b^\varepsilon+c^\varepsilon)^k
$$
ทางซ้ายมือมี 3 ทั้งหมด $k-2$ ตัว และเขียน $3=1+1+1$ เวลาใช้อสมการของ Holder

จากอสมการข้างต้นได้ว่า
$$
\frac{a^{\varepsilon k}}{b+1}+\frac{b^{\varepsilon k}}{c+1}
+\frac{c^{\varepsilon k}}{a+1}\geq(a^\varepsilon+b^\varepsilon+c^\varepsilon)^{k}/(2\times3^{k-1})
$$
เนื่องจาก
$$
\left(\frac{1}{a}+\frac{1}{b}+\frac{1}{c}\right)(a+b+c)\geq3^2
$$
ดังนั้น $\sum a^\varepsilon\geq3$ เพราะฉะนั้นเราได้อสมการตามต้องการ

หวัดดีปีใหม่ครับพ่อแม่พี่น้องชาว Mathcenter ทุกทั่น ขอให้ทุกคนมีความสุขในปีใหม่นี้ครับ

14 มกราคม 2006 11:45 : ข้อความนี้ถูกแก้ไขแล้ว 4 ครั้ง, ครั้งล่าสุดโดยคุณ Punk
ตอบพร้อมอ้างอิงข้อความนี้
  #23  
Old 15 มกราคม 2006, 01:58
warut warut ไม่อยู่ในระบบ
กระบี่ไร้สภาพ
 
วันที่สมัครสมาชิก: 24 พฤศจิกายน 2001
ข้อความ: 1,627
warut is on a distinguished road
Smile

ด้วยความช่วยเหลือของคุณ TOP ในที่สุดผมก็พบว่าโจทย์ข้อ 9 ของคุณ nooonuii นั้น เคยมีการถามมาครั้งนึงแล้วโดยคุณ nooonuii เอง และในครั้งนั้นก็เป็นข้อ 9 เหมือนกัน ส่วนคนที่มาตอบก็คือคุณ M@gpie เหมือนครั้งนี้เปี๊ยบเลย แปลกดี

18 พฤษภาคม 2007 14:51 : ข้อความนี้ถูกแก้ไขแล้ว 1 ครั้ง, ครั้งล่าสุดโดยคุณ mathcenter
เหตุผล: Tag Post
ตอบพร้อมอ้างอิงข้อความนี้
  #24  
Old 15 มกราคม 2006, 04:24
nooonuii nooonuii ไม่อยู่ในระบบ
ผู้พิทักษ์กฎทั่วไป
 
วันที่สมัครสมาชิก: 25 พฤษภาคม 2001
ข้อความ: 6,408
nooonuii is on a distinguished road
Post

อ้างอิง:
ข้อความเดิมของคุณ warut:
ด้วยความช่วยเหลือของคุณ TOP ในที่สุดผมก็พบว่าโจทย์ข้อ 9 ของคุณ nooonuii นั้น เคยมีการถามมาครั้งนึงแล้วโดยคุณ nooonuii เอง และในครั้งนั้นก็เป็นข้อ 9 เหมือนกัน ส่วนคนที่มาตอบก็คือคุณ M@gpie เหมือนครั้งนี้เปี๊ยบเลย แปลกดี
อะจึ๋ย จริงหรือครับคุณ Warut แปลกจริงๆด้วยครับ เพราะว่าผมก็ไม่ได้ทำเครื่องหมายไว้ด้วยว่าโจทย์ข้อไหนเคยเอามาถามแล้วบ้าง ก็เลยสุ่มๆเอา ช่วงนี้ยุ่งจังเลยครับ เลยมาเฉลยได้ทีละข้อสองข้อ แต่ครบแน่นอนครับ
__________________
site:mathcenter.net คำค้น

18 พฤษภาคม 2007 14:52 : ข้อความนี้ถูกแก้ไขแล้ว 1 ครั้ง, ครั้งล่าสุดโดยคุณ mathcenter
เหตุผล: Tag Post
ตอบพร้อมอ้างอิงข้อความนี้
  #25  
Old 15 มกราคม 2006, 06:23
nooonuii nooonuii ไม่อยู่ในระบบ
ผู้พิทักษ์กฎทั่วไป
 
วันที่สมัครสมาชิก: 25 พฤษภาคม 2001
ข้อความ: 6,408
nooonuii is on a distinguished road
Post

2.(อสมการ) ให้ a,b,c > 0 โดยที่ a + b + c = 3 จงพิสูจน์ว่า

\[ \frac{a^n}{b+1} + \frac{b^n}{c+1} + \frac{c^n}{a+1} \geq \frac{3}{2} \]

ทุกจำนวนเต็ม n
----------------------------------------------------------------------------------------------------
ข้อนี้ยากไปนิดนึงครับเพราะผม generalize มาจากโจทย์ในวารสาร Crux
My Solution : ก่อนอื่นจะพิสูจน์ว่า
\[ a^n + b^n + c^n\geq 3 \]
ทุกค่า n

Proof :
1) ถ้า n = 0 สมการเป็นจริง
2) ถ้า n1 จะได้ว่า
\[ \displaystyle{ \frac{a^n + b^n + c^n}{3}\geq (\frac{a + b + c}{3})^n = 1 } \]
3) ถ้า n -1 จะได้ว่า
\[ \displaystyle{ \frac{(\frac{1}{a})^{-n} + (\frac{1}{b})^{-n} + (\frac{1}{c})^{-n}}{3} \geq (\frac{\frac{1}{a} + \frac{1}{b} + \frac{1}{c} }{3})^{-n} \geq (\frac{3}{a+b+c})^{-n} = 1 } \]

Proof of Inequality :

1) n เป็นจำนวนคู่จะได้ว่า

\( \displaystyle{ 3 \leq a^{\frac{n}{2}} + b^{\frac{n}{2}} + c^{\frac{n}{2}} } \)
\( \displaystyle{ = \frac{a^{\frac{n}{2}}}{\sqrt{b+1}}\cdot\sqrt{ b+1 } + \frac{b^{\frac{n}{2}}}{\sqrt{c+1}}\cdot\sqrt{ c+1 } + \frac{c^{\frac{n}{2}}}{\sqrt{a+1}}\cdot\sqrt{ a+1 } } \)
\( \displaystyle{ \leq \sqrt{\frac{a^n}{b+1} + \frac{b^n}{c+1} + \frac{c^n}{a+1}} \cdot \sqrt{a + b + c + 3} } \)
\( \displaystyle{ = \sqrt{6}\sqrt{\frac{a^n}{b+1} + \frac{b^n}{c+1} + \frac{c^n}{a+1}} } \)

2) n เป็นจำนวนคี่จะได้

\( \displaystyle{ 3 \leq a^{\frac{n+1}{2}} + b^{\frac{n+1}{2}} + c^{\frac{n+1}{2}} } \)
\( \displaystyle{ = \frac{a^{\frac{n}{2}}}{\sqrt{b+1}}\cdot\sqrt{ a(b+1) } + \frac{b^{\frac{n}{2}}}{\sqrt{c+1}}\cdot\sqrt{ b(c+1) } + \frac{c^{\frac{n}{2}}}{\sqrt{a+1}}\cdot\sqrt{ c(a+1) } } \)
\( \displaystyle{ \leq \sqrt{\frac{a^n}{b+1} + \frac{b^n}{c+1} + \frac{c^n}{a+1}} \cdot \sqrt{ab + bc + ca + a+b+c} } \)
\( \displaystyle{ \leq \sqrt{6}\sqrt{\frac{a^n}{b+1} + \frac{b^n}{c+1} + \frac{c^n}{a+1}} } \)

อสมการสุดท้ายมาจาก
\( \displaystyle{ ab+bc+ca \leq \frac{(a+b+c)^2}{3} = 3 } \)

เฮ้อเหนื่อยโจทย์ข้อนี้ใช้ Latex กันหัวบานเลยเหอเหอ
__________________
site:mathcenter.net คำค้น
ตอบพร้อมอ้างอิงข้อความนี้
  #26  
Old 15 มกราคม 2006, 06:43
nooonuii nooonuii ไม่อยู่ในระบบ
ผู้พิทักษ์กฎทั่วไป
 
วันที่สมัครสมาชิก: 25 พฤษภาคม 2001
ข้อความ: 6,408
nooonuii is on a distinguished road
Post

ข้อ 3 แป้กครับ ไม่คิดว่ามันจะง่ายไป ลืมไปว่าที่นี่มีเซียนทฤษฎีจำนวนอย่างคุณ Warut อยู่ เหอเหอ คราวหน้าขอแก้ตัวใหม่ครับ

ข้อ 4 เหมือนคุณ Warut ทุกประการครับ

ข้อ 5 เป็นกรณีเฉพาะของทฤษฎีที่คุณ Warut ใช้แก้โจทย์ข้อนี้นั่นเองครับ แต่สามารถพิสูจน์ได้โดยใช้แค่ความรู้ของเซตและการแก้ระบบสมการเชิงเส้น

ข้อ 6 ก็เหมือนคุณ Warut ทุกประการเลยครับ สงสัยจะโดนคุณ Warut จับไต๋ได้ซะแล้วอิอิ
__________________
site:mathcenter.net คำค้น
ตอบพร้อมอ้างอิงข้อความนี้
  #27  
Old 16 มกราคม 2006, 00:35
nooonuii nooonuii ไม่อยู่ในระบบ
ผู้พิทักษ์กฎทั่วไป
 
วันที่สมัครสมาชิก: 25 พฤษภาคม 2001
ข้อความ: 6,408
nooonuii is on a distinguished road
Post

7.(พีชคณิตของพหุนาม) ให้ P(z) เป็นพหุนามในจำนวนเชิงซ้อนซึ่ง |Re(P(z))| = |Im(P(z))| ทุกค่า z จงพิสูจน์ว่า มีจำนวนจริง a ซึ่งทำให้

\[ [P(z)]^4 + 4a^4 = 0 \]

ทุกค่า z

ข้อนี้ผมว่าคำตอบของคุณ nongtum ยังไม่ตรงกับวัตุประสงค์ของโจทย์นะครับเพราะว่า ค่า a ที่คุณ nongtum ได้มามันขึ้นอยู่กับค่า z แต่ละตัวครับ แต่โจทย์ต้องการค่า a ซึ่งใช้ได้กับทุก z ซึ่งจริงๆแล้วโจทย์ต้องการให้เราพิสูจน์ว่า P(z) เป็นพหุนามคงตัวนั่นเอง

My Solution : โดยทฤษฎีบทหลักมูลพีชคณิตเราจะได้ว่า ถ้า P(z) ไม่ใช่พหุนามคงตัวแล้ว P(z) จะเป็นฟังก์ชันทั่วถึงบน \( \mathbb{C}\) แต่จากเงื่อนไขโจทย์เราได้ว่า
\[ P(\mathbb{C}) \subseteq \{ x+iy : |x|=|y| \} \neq \mathbb{C} \]

ดังนั้น P(z) เป็นพหุนามคงตัว นั่นคือ P(z) = a + bi , |a| = |b| ทุกค่า z
ดังนั้น
\[ [P(z)]^4 = (a+bi)^4 = (2abi)^2 = -4a^4 \]
ทุกค่า z
__________________
site:mathcenter.net คำค้น
ตอบพร้อมอ้างอิงข้อความนี้
  #28  
Old 16 มกราคม 2006, 05:23
warut warut ไม่อยู่ในระบบ
กระบี่ไร้สภาพ
 
วันที่สมัครสมาชิก: 24 พฤศจิกายน 2001
ข้อความ: 1,627
warut is on a distinguished road
Smile

อ้างอิง:
ข้อความเดิมของคุณ nooonuii:
ข้อ 3 แป้กครับ
นี่แหละ...ลูกผู้ชายตัวจริง
อ้างอิง:
ข้อความเดิมของคุณ nooonuii:
ข้อ 5 เป็นกรณีเฉพาะของทฤษฎีที่คุณ Warut ใช้แก้โจทย์ข้อนี้นั่นเองครับ แต่สามารถพิสูจน์ได้โดยใช้แค่ความรู้ของเซตและการแก้ระบบสมการเชิงเส้น
ถ้างั้นว่างๆคุณ nooonuii ช่วยแสดงให้ดูด้วยนะครับ

ส่วนข้อ 7 นี่ผมก็เพิ่งมาเข้าใจตอนอ่านเฉลยนี่แหละ ตอนนู้นก็คล้อยตามไปกับคุณ nongtum ด้วยเหมือนกันครับ
ตอบพร้อมอ้างอิงข้อความนี้
  #29  
Old 16 มกราคม 2006, 05:52
nongtum's Avatar
nongtum nongtum ไม่อยู่ในระบบ
ผู้พิทักษ์กฎทั่วไป
 
วันที่สมัครสมาชิก: 10 เมษายน 2005
ข้อความ: 3,246
nongtum is on a distinguished road
Post

แหะๆๆ ข้อ 7 ผมอ่านโจทย์เข้าใจผิดไปจริงๆแหละครับ ทำแบบก้าวกระโดดไปนิด ถึงว่าทำไมมันดูง่ายผิดปกติ (เหมือนโดนอาจารย์ติงมายังไงยังงั้น)

ส่วนข้อพิเศษของคุณ m@gpie ต้องขอโทษคุณ warut และคุณ m@gpie ด้วยครับที่ตอบแบบเหมารวม เพราะลองมาเช็คดูอีกที $s^5−s^4+s^2−s+1$ ใช้ Eisenstein ตรวจสอบไม่ได้ครับ ไว้ว่างๆจะลองใช้วิธือื่นตรวจสอบดูครับ
__________________
คนไทยร่วมใจอย่าใช้ภาษาวิบัติ
ฝึกพิมพ์สัญลักษณ์สักนิด ชีวิต(คนตอบและคนถาม)จะง่ายขึ้นเยอะ (จริงๆนะ)

Stay Hungry. Stay Foolish.
ตอบพร้อมอ้างอิงข้อความนี้
  #30  
Old 16 มกราคม 2006, 12:18
nooonuii nooonuii ไม่อยู่ในระบบ
ผู้พิทักษ์กฎทั่วไป
 
วันที่สมัครสมาชิก: 25 พฤษภาคม 2001
ข้อความ: 6,408
nooonuii is on a distinguished road
Post

ผมว่าวิธีตรวจสอบ irreducibility ของพหุนาม $s^5−s^4+s^2−s+1$ คงต้องใช้วิธีหาข้อขัดแย้งแล้วล่ะครับ ซึ่งโดย Rational Root Test เราจะได้ว่าพหุนามนี้ไม่มีรากที่เป็นจำนวนตรรกยะ ดังนั้น ถ้ามัน reducible มันจะอยู่ในรูปผลคูณของพหุนามกำลังสองและกำลังสาม จากนั้นเทียบสัมประสิทธิ์ก็จะได้ข้อขัดแย้งครับ แต่ก็ถึกน่าดูชม

ป.ล. โจทย์ข้อ 7 ยังคงเป็นจริงถ้าเราเปลี่ยนจากพหุนามเป็น entire function การพิสูจน์จะง่ายมากถ้าใช้ Picard's Little Theorem ครับ
__________________
site:mathcenter.net คำค้น

16 มกราคม 2006 12:35 : ข้อความนี้ถูกแก้ไขแล้ว 1 ครั้ง, ครั้งล่าสุดโดยคุณ nooonuii
ตอบพร้อมอ้างอิงข้อความนี้
ตั้งหัวข้อใหม่ Reply


หัวข้อคล้ายคลึงกัน
หัวข้อ ผู้ตั้งหัวข้อ ห้อง คำตอบ ข้อความล่าสุด
ใครรู้จัก NP-Problem มั่งครับ ช่วยเข้ามาคุยกันหน่อย fangolf ปัญหาคณิตศาสตร์ทั่วไป 0 05 กุมภาพันธ์ 2007 10:10
LQR Problem M@gpie ปัญหาคณิตศาสตร์ทั่วไป 0 24 กันยายน 2006 16:50
ปัญหาชิงรางวัลข้อที่ 2: Log Problem warut คณิตศาสตร์อุดมศึกษา 8 16 มกราคม 2006 05:04
set problem brother ปัญหาคณิตศาสตร์ทั่วไป 1 11 เมษายน 2005 02:06
HaPpY NeW YeAr กานหน่อยคร้าบบบ จะปีใหม่แล้ว M@gpie ปัญหาคณิตศาสตร์ทั่วไป 4 01 มกราคม 2004 16:31


กฎการส่งข้อความ
คุณ ไม่สามารถ ตั้งหัวข้อใหม่ได้
คุณ ไม่สามารถ ตอบหัวข้อได้
คุณ ไม่สามารถ แนบไฟล์และเอกสารได้
คุณ ไม่สามารถ แก้ไขข้อความของคุณเองได้

vB code is On
Smilies are On
[IMG] code is On
HTML code is Off
ทางลัดสู่ห้อง


เวลาที่แสดงทั้งหมด เป็นเวลาที่ประเทศไทย (GMT +7) ขณะนี้เป็นเวลา 12:35


Powered by vBulletin® Copyright ©2000 - 2024, Jelsoft Enterprises Ltd.
Modified by Jetsada Karnpracha